LSAT and Law School Admissions Forum

Get expert LSAT preparation and law school admissions advice from PowerScore Test Preparation.

User avatar
 Dave Killoran
PowerScore Staff
  • PowerScore Staff
  • Posts: 5850
  • Joined: Mar 25, 2011
|
#72722
This game is also discussed in our Podcast, at the 48:04 mark: LSAT Podcast Episode 37: The November 2019 LSAT Logic Games Section

Complete Question Explanation

The correct answer choice is (A).

Answer choice (A): This is the correct answer choice, and this is a viable solution to the F and G rows.

Answer choice (B): Per the first rule, V's visit to Ghana must be in the second or third month. This answer, which places V's visit fourth, thus cannot occur.

Answer choice (C): With W, Y, and Z each making visits to France in this answer, only V remains to make all four India visits. Since that is impossible, this answer cannot occur.

Answer choice (D): This answer violates Rule #4, which stipulates that Y cannot make a visit in the fourth month.

Answer choice (E): This answer violates Rule #3, which stipulates that in the Ghana row there must be a YW block. The two variables are instead reversed.

Get the most out of your LSAT Prep Plus subscription.

Analyze and track your performance with our Testing and Analytics Package.